Explanation for Eliminating B Please
Hi In terms of answer-choice B, can someone please offer an explanation for eliminating it oth...
Mazen on July 29, 2022
  • December 2000 LSAT
  • SEC3
  • Q9
2
Replies
How do you negate C?
Sometimes, the negation gets funky and outta hand man. That "unless" screws everything up, plus y...
Tyler808 on July 16, 2022
  • December 2000 LSAT
  • SEC3
  • Q11
1
Reply
Please help
Could someone please explain why B is wrong and C is correct?
yckim2180 on May 30, 2021
  • December 2000 LSAT
  • SEC3
  • Q6
1
Reply
Help
This is a strengthen with sufficient premise question, isn't it? Then, how does a choice (A) stre...
Batman on April 11, 2021
  • December 2000 LSAT
  • SEC3
  • Q21
8
Replies
Can someone please explain this question and go...
HELP!
cheynnelee on January 28, 2021
  • December 2000 LSAT
  • SEC3
  • Q9
1
Reply
A vs D
Hello, I was so torn between A and D. A makes sense because the craft was being flown for "goo...
cheynnelee on January 28, 2021
  • December 2000 LSAT
  • SEC3
  • Q15
1
Reply
Clarification of B
Is B incorrect because it is more of what the stim is trying to establish rather than an assumpti...
cheynnelee on January 28, 2021
  • December 2000 LSAT
  • SEC3
  • Q16
1
Reply
Please explain this question!!
I am having major difficulty seeing why this answer is correct. Raphaela does not mention emigrat...
cheynnelee on January 28, 2021
  • December 2000 LSAT
  • SEC3
  • Q19
1
Reply
Help
Could someone please explain why the answer is C and not B?
yckim2180 on January 7, 2021
  • December 2000 LSAT
  • SEC3
  • Q6
1
Reply
explain
Can someone explain this question and each answer choice?
priyap59 on December 23, 2020
  • December 2000 LSAT
  • SEC3
  • Q7
1
Reply
Must Be True
I was stuck between A and C here - and ultimately (and incorrectly) chose C. I thought that C mat...
Anna20 on August 7, 2020
  • December 2000 LSAT
  • SEC3
  • Q8
1
Reply
Is this an illustration question?
Is this an illustration question?
TheFacu on November 23, 2019
  • December 2000 LSAT
  • SEC3
  • Q8
3
Replies
Explain!
Could someone explain to me why e is wrong and d is right? Is the government mentioned in the sti...
Ashley-Tien-2 on October 25, 2018
  • December 2000 LSAT
  • SEC3
  • Q16
1
Reply
Argument
Is this the correct way to think about the stimulus? There is a premise stating a general rule an...
Ashley-Tien on July 3, 2018
  • December 2000 LSAT
  • SEC3
  • Q22
1
Reply